Finding Volume of Solid in First Octant with Bounded Polar Equations

  • Thread starter Thread starter andromeda92
  • Start date Start date
  • Tags Tags
    Solid Volume
Click For Summary
The discussion revolves around finding the volume of a solid in the first octant bounded by the equations r=1+cos(θ), z=y, and z=0. The user is attempting to set up the double integral for volume but is uncertain about the limits of integration and the complexity of the resulting expressions. They have converted the z equations into polar coordinates, leading to an integral that appears complicated due to the presence of a trigonometric function raised to the third power. Participants suggest considering the limits for θ based on the first quadrant and whether to use cylindrical or spherical coordinates for simplification. The conversation highlights the challenges of integrating in polar coordinates and the need for clarity in setting up the problem correctly.
andromeda92
Messages
1
Reaction score
0

Homework Statement



find the volume of the solid bounded by the graphs of the given equations:

r=1+cos∅
z=y
z=0
(the first octant)

Homework Equations



V=\int\int rdrd∅


The Attempt at a Solution



So, I've been having trouble deciding what to integrate from and to. I converted the z equations into polar coordinates, so, z=rsin and z=0

\int^{2\pi}_{0}\int^{1+cos∅}_{0} (rsin)rdrd∅

Mod note: revised LaTeX
\int_0^{2\pi} \int_0^{1 + cos(\theta)} rsin(\theta) r~dr~d\theta

I found, from the first integration, [2∏]\int[/0]r^(3)sin∅/3 d∅ and I need to plug in (1+cos∅) and 0 for r...but, this seems WAY too complicated having a trig function to the 3rd power.<br /> <br /> I&#039;m not even sure if I have the beginning correct. Can anyone help me out?
 
Last edited by a moderator:
Physics news on Phys.org
In three dimensions, the first quadrant has x ≥ 0, y ≥ 0, z ≥ 0, and think about the limits of ∅ for x ≥ 0, y ≥ 0.

Think about this - http://www.wolframalpha.com/input/?i=polar+plot+r=1+cos+theta (don't worry about the angle being theta)

And do you want to use cylindrical or spherical coordinates?
 
Last edited:
Question: A clock's minute hand has length 4 and its hour hand has length 3. What is the distance between the tips at the moment when it is increasing most rapidly?(Putnam Exam Question) Answer: Making assumption that both the hands moves at constant angular velocities, the answer is ## \sqrt{7} .## But don't you think this assumption is somewhat doubtful and wrong?

Similar threads

  • · Replies 19 ·
Replies
19
Views
2K
  • · Replies 20 ·
Replies
20
Views
2K
  • · Replies 6 ·
Replies
6
Views
2K
  • · Replies 9 ·
Replies
9
Views
2K
Replies
3
Views
2K
  • · Replies 2 ·
Replies
2
Views
2K
  • · Replies 4 ·
Replies
4
Views
1K
  • · Replies 4 ·
Replies
4
Views
2K
  • · Replies 3 ·
Replies
3
Views
2K
Replies
14
Views
2K